Saturday 2 April 2016

quantum field theory - Does Peskin & Schroeder Eq. (4.26), $U(t_1,t_2)U(t_2,t_3) = U(t_1,t_3)$ imply $[H_0,H_{int}] = 0$?


Peskin & Schroeder equation (4.17) define the operator, \begin{equation} U(t,t_{0})~=~e^{i(t-t_{0})H_{0}}e^{-i(t-t_{0})H} \tag{4.17} \end{equation} where $$H~=~H_0+H_{\text{int}}\tag{4.12}$$ is the full Hamiltonian and $H_{0}$ is the free Hamilton, both in the Schrodinger picture. In equation (4.26), Peskin and Schroeder state that the operator satisfies the following identity, \begin{equation} U(t_{1},t_{2})U(t_{2},t_{3})~=~U(t_{1},t_{3}) \tag{4.26} \end{equation} where $t_{1}\ge t_{2}\ge t_{3}$. Does this imply that the free Hamiltonian commutes with the interaction $$[H_{0},H_{\text{int}}]~=~0~ ?$$ Here is my argument that it does.


In the condition $t_{1}\ge t_{2}\ge t_{3}$ take $t_{2}=0$. The identity is then, \begin{equation} U(t_{1},0)U(0,t_{3})=U(t_{1},t_{3})\ . \end{equation} Substitute the definition, \begin{equation} e^{it_{1}H_{0}}e^{-it_{1}H}e^{-it_{3}H_{0}}e^{it_{3}H}=e^{i(t_{1}-t_{3})H_{0}}e^{-i(t_{1}-t_{3})H} \end{equation} and simplify to get, \begin{equation} e^{-it_{1}H}e^{-it_{3}H_{0}}=e^{-it_{3}H_{0}}e^{-it_{1}H} \end{equation} with $t_{1}\ge 0\ge t_{3}$ . Put $t_{1}=t$ and $t_{3}=-t$. \begin{equation} e^{-itH}e^{itH_{0}}=e^{itH_{0}}e^{-itH} \end{equation} Expanding to second order in $t$, \begin{equation} (1-itH-\frac{t^{2}}{2}HH)(1+itH_{0}-\frac{t^{2}}{2}H_{0}H_{0})= (1+itH_{0}-\frac{t^{2}}{2}H_{0}H_{0})(1-itH-\frac{t^{2}}{2}HH) \end{equation} results in, \begin{equation} HH_{0}=H_{0}H \end{equation} so that $[H_{0},H]_{-}=0$. Now $H=H_{0}+H_{int}$ so the free Hamiltonian must commute with the interaction. \begin{equation} [H_{0},H_{int}]_{-}=0 \end{equation} In Peskin and Schroeder, the context for this material is the self-interacting scalar field with Hamiltonian, \begin{equation} H=\int d^{3}x \left(\frac{1}{2}\pi(t,x)^{2}+\frac{1}{2}\frac{\partial \phi}{\partial x^{r}}\frac{\partial \phi}{\partial x^{r}}+V(\phi)\right) \ . \end{equation} In the classical theory, the PB is, \begin{equation} [H_{0},H_{int}]_{PB}=-\int d^{3}x\frac{\delta H_{0}}{\delta \pi}\frac{\delta H_{int}}{\delta \phi}=-\int d^{3}x\ \pi\frac{dV}{d\phi}=-\frac{d}{dt}\int d^{3}x\ V(\phi(t,x)) \end{equation} Going over to quantum theory, \begin{equation} [H_{0},H_{int}]_{-}=-i\frac{d}{dt}\int d^{3}x\ V(\phi(t,x)) \end{equation} so that $[H_{0},H_{int}]_{-}=0\ $ implies the integral of $V(\phi)$ is a conserved charge; is this also a correct result?



Answer



Ref. 1 writes the correct formula



$$ U(t,t^{\prime})~=~e^{iH_0(t-t_0)} e^{-iH(t-t^{\prime})}e^{-iH_0(t^{\prime}-t_0)} , \qquad t~\geq~ t^{\prime},\tag{4.25}$$


which satisfies


$$ U(t_1,t_2)U(t_2,t_3)~=~U(t_1,t_3) , \qquad t_1~\geq~ t_2~\geq~ t_3.\tag{4.26}$$


Here $t_0$ is an arbitrary but fixed fiducial initial instant where operators and states in the Schrödinger picture, the Heisenberg picture and the interaction picture all agree. For $t\neq t_0$, the three pictures are no longer the same, although they are still unitary equivalent.


For $t^{\prime}=t_0$, eq. (4.25) simplifies to


$$ U(t,t_0)~=~e^{iH_0(t-t_0)}e^{-iH(t-t_0)}. \tag{4.17}$$


It appears that OP mistakenly replaces $t_0$ in eq. (4.17) with an arbitrary time $t^{\prime} \leq t$. The resulting equation


$$ U(t,t^{\prime})~=~e^{iH_0(t-t^{\prime})}e^{-iH(t-t^{\prime})}. \qquad(\leftarrow \text{Wrong!})$$


is not correct.


References:




  1. M.E. Peskin & D.V. Schroeder, An Intro to QFT; Section 4.2.


No comments:

Post a Comment

Understanding Stagnation point in pitot fluid

What is stagnation point in fluid mechanics. At the open end of the pitot tube the velocity of the fluid becomes zero.But that should result...